Đến nội dung

Hoang Tung 126 nội dung

Có 1000 mục bởi Hoang Tung 126 (Tìm giới hạn từ 09-06-2020)



Sắp theo                Sắp xếp  

#482128 $\boxed{\text{Chuyên Đề}}$ Bất đẳng thức - Cực trị

Đã gửi bởi Hoang Tung 126 on 09-02-2014 - 09:36 trong Bất đẳng thức và cực trị

Bài 30:Áp dụng bđt $x^5+y^5\geq \frac{(x+y)^5}{16}$

$= > \sum \sqrt[5]{a^5+b^5}\geq \sum \sqrt[5]{\frac{(a+b)^5}{16}}=\sum \frac{a+b}{\sqrt[5]{16}}=\sqrt[5]{2}\sum a$




#482917 $\boxed{\text{Chuyên Đề}}$ Bất đẳng thức - Cực trị

Đã gửi bởi Hoang Tung 126 on 13-02-2014 - 18:29 trong Bất đẳng thức và cực trị

52) Cho $\left\{\begin{matrix}a;b;c>0 & & \\ a+b+c=4 & & \end{matrix}\right.$. Cmr: $4\leq \sum \sqrt{a+b}\leq 2\sqrt{6}$

 

53) Cho $a;b;c$ là độ dài 3 cạnh một tam giác, Cmr: $\sum \sqrt{a+b-c}\leq \sum \sqrt{a}$

 

54) Cho $a;b;c$ là độ dài 3 cạnh một tam giác có chu vi bằng $2$. Cmr: $\sum a^2<2(1-abc)$

 

55) Xác định dạng tam giác $ABC$ có chu vi bằng $1$ biết $A=\sum \frac{a}{1-2a}$ đạt $min$.

 

56) Cho $a;b;c>0$ thỏa $a^2+b^2+c^2=1$. Cmr: $\sum \frac{a^2}{1+b-a}\geq 1$

 

57) Cho $a;b;c$ là độ dài 3 cạnh một tam giác. Cmr: $\sum \sqrt[3]{a+b-c}\leq \sum \sqrt[3]{a}$

Bài 56:Theo Bunhia có:$\sum \frac{a^2}{1+b-a}=\sum \frac{a^4}{a^2+a^2b-a^3}\geq \frac{(\sum a^2)^2}{\sum a^2+\sum a^2b-\sum a^3}\geq \sum a^2=1$

(Do áp dụng AM-GM 3 số có:$\sum a^3\geq \sum a^2b$)




#482914 $\boxed{\text{Chuyên Đề}}$ Bất đẳng thức - Cực trị

Đã gửi bởi Hoang Tung 126 on 13-02-2014 - 18:27 trong Bất đẳng thức và cực trị

52) Cho $\left\{\begin{matrix}a;b;c>0 & & \\ a+b+c=4 & & \end{matrix}\right.$. Cmr: $4\leq \sum \sqrt{a+b}\leq 2\sqrt{6}$

 

53) Cho $a;b;c$ là độ dài 3 cạnh một tam giác, Cmr: $\sum \sqrt{a+b-c}\leq \sum \sqrt{a}$

 

54) Cho $a;b;c$ là độ dài 3 cạnh một tam giác có chu vi bằng $2$. Cmr: $\sum a^2<2(1-abc)$

 

55) Xác định dạng tam giác $ABC$ có chu vi bằng $1$ biết $A=\sum \frac{a}{1-2a}$ đạt $min$.

 

56) Cho $a;b;c>0$ thỏa $a^2+b^2+c^2=1$. Cmr: $\sum \frac{a^2}{1+b-a}\geq 1$

 

57) Cho $a;b;c$ là độ dài 3 cạnh một tam giác. Cmr: $\sum \sqrt[3]{a+b-c}\leq \sum \sqrt[3]{a}$

Bài 55:Ta có:$\sum \frac{a}{1-2a}=\sum \frac{a^2}{a-2a^2}\geq \frac{(\sum a)^2}{\sum a-2\sum a^2}=\frac{1}{1-2\sum a^2}\geq \frac{1}{1-2.\frac{(\sum a)^2}{3}}=\frac{1}{1-\frac{2}{3}}=3$




#483035 $\boxed{\text{Chuyên Đề}}$ Bất đẳng thức - Cực trị

Đã gửi bởi Hoang Tung 126 on 14-02-2014 - 08:32 trong Bất đẳng thức và cực trị

58) Cho $x\in (0;1)$. Tìm Min $A=\frac{4x^2+1}{x^2(1-x)}$

 

59) Cho $x\in (0;3)$. Tìm Max $B=(5x^2-14x-3)(x-3)$

 

60) Cho $a;b;c$ là độ dài 3 cạnh một tam giác. Cmr: $\sum \frac{a}{\sqrt{b+c-a}}\geq \sum \sqrt{a}$

 

61) Cho $a;b;c$ là độ dài 3 cạnh một tam giác. Cmr: $|\sum \frac{a-b}{a+b}|<\frac{1}{8}$

 

62) Cho $x\in [0;1]$. Cmr: $\sqrt[4]{1-x^2}+\sqrt[4]{1-x}+\sqrt[4]{1+x}\leq 3$

 

63) Cho $a;b;c>0$. Cmr: $\frac{1}{a^3}+\frac{a^3}{b^3}+b^3\geq \frac{1}{a}+\frac{a}{b}+b$

 

64)

a) Cho $x\in [-1;1]$. Cmr: $|4x^3-3x|\leq 1$

b) Cho $\left\{\begin{matrix}a_1;a_2;...;a_n\in [-1;1] & & \\ a_1^3+a_2^3+...+a_n^3=0 & & \end{matrix}\right.$. Cmr: $a_1+a_2+...+a_n\leq \frac{n}{3}$

 

65) Cho $x;y>0$ thỏa $x^2+y^2=1$. Cmr: $xy+Max(x;y)\leq \frac{3\sqrt{3}}{4}$ (Giải thích: Nếu $x>y$ thì $Max(x;y)=x$ và tương tự)

 

 

P/s: Anh Daicagiangho1998 học KHTN nên cứ từ từ mà làm thôi, chứ có đề phát đã ăn hết sạch luôn vậy :D

Bài 65:Bài này khá hay:Giả sử $x=Max(x,y)$

 Theo AM-GM có:$xy\leq \frac{x^2+3y^2}{2\sqrt{3}},x\leq \frac{4x^2+3}{4\sqrt{3}}$

$= > xy+max(x,y)=x+y\leq \frac{2(x^2+3y^2)+(4x^2+3)}{4\sqrt{3}}=\frac{6(x^2+y^2)+3}{4\sqrt{3}}=\frac{3\sqrt{3}}{4}$

Dấu = xảy ra tại $x=\frac{\sqrt{3}}{2},y=\frac{1}{2}$




#484146 $\boxed{\text{Chuyên Đề}}$ Bất đẳng thức - Cực trị

Đã gửi bởi Hoang Tung 126 on 22-02-2014 - 17:26 trong Bất đẳng thức và cực trị

Đăng bài đi Việt Hoàng




#495292 $\boxed{\text{Chuyên Đề}}$ Bất đẳng thức - Cực trị

Đã gửi bởi Hoang Tung 126 on 26-04-2014 - 19:35 trong Bất đẳng thức và cực trị

Bài 153: 
Cho $a,b,c\geq 0$ thỏa $a+b+c=3$. Tìm GTLN của: 
$S=(a^2-ab+b^2)(b^2-bc+c^2)(c^2-ca+a^2)$

Bài này chưa ai giải thì xơi vậy...

Gỉa sử $a\geq b\geq c= > b^2-bc+c^2=b^2+c(c-b)\leq b^2,c^2-ac+a^2=a^2+c(c-a)\leq a^2$

$= > S\leq a^2b^2(a^2-ab+b^2)=\frac{4}{9}.\frac{3ab}{2}.\frac{3ab}{2}(a^2-ab+b^2)\leq \frac{4}{9}.\frac{(\frac{3ab}{2}+\frac{3ab}{2}+a^2-ab+b^2)^3}{27}=\frac{4(a+b)^6}{9.27}\leq \frac{4(a+b+c)^6}{9.27}=\frac{4.3^6}{9.27}=12= > S\leq 12$

Dấu = xảy ra khi $c=0,a=1,b=2$

 

Bài này em đưa link giải rồi Hì :)

 


 

Ở đây

 

 




#493516 $\boxed{\text{Chuyên Đề}}$ Bất đẳng thức - Cực trị

Đã gửi bởi Hoang Tung 126 on 17-04-2014 - 16:30 trong Bất đẳng thức và cực trị

$144$,(Tự sáng tác ^^) Với $a,b,c>0$ tmđk $3a+5b+8c=1$

CMR; $(1-a)^{3}(1-b)^{5}(1-c)^{8} \geqslant 15^{16}a^{3}b^{5}c^{8}$ 

 

@Viet Hoang 99: Chú ý không kẹp $$ vào trong tiếng Việt có dấu.

Ta có :$(1-a)^3(1-b)^5(1-c)^8=(3a+5b+8c-a)^3(3a+5b+8c-b)^5(3a+5b+8c-c)^8=(2a+5b+8c)^3(3a+4b+8c)^5(3a+5b+7c)^8$

Theo Cosi thì $(2a+5b+8c)^3(3a+4b+8c)^5(3a+5b+7c)^8\geq (15\sqrt[15]{a^2b^5c^8})^3(15\sqrt[15]{a^3b^4c^8})^5(15\sqrt[15]{a^3b^5c^7})^8=(15^{16})(a^3b^5c^8)$

Dấu = khi $a=b=c=\frac{1}{16}$




#491603 $\boxed{\text{Chuyên Đề}}$ Bất đẳng thức - Cực trị

Đã gửi bởi Hoang Tung 126 on 09-04-2014 - 11:09 trong Bất đẳng thức và cực trị

Bài 168: Cho $a,b,c$ thỏa mãn:$a^2+b^2+c^2=1$.

Tìm GTLN của biểu thức: $A=(a-b)(b-c)(c-a)(a+b+c)$

 

Bài 169: Cho các số thực $a,b,c$. Chứng minh rằng:

$\sum \sqrt{\frac{a(b+c)}{a^2+bc}}\leq \sqrt{\sum \sqrt{a}.\sum \frac{1}{\sqrt{a}}}$

Bài 169:Ta có:$(\sum \sqrt{a})(\sum \frac{1}{\sqrt{a}})\geq 3\sqrt[3]{\sqrt{abc}}.3\sqrt[3]{\frac{1}{\sqrt{abc}}}=9= > \sqrt{(\sum \sqrt{a})(\sum \frac{1}{\sqrt{a}})}\geq 3$

Mà $\sum \sqrt{\frac{a(b+c)}{a^2+bc}}\leq \sqrt{3(\sum \frac{a(b+c)}{a^2+bc})}\leq 3< = = > \sum \frac{a(b+c)}{a^2+bc}\leq 3< = > \sum (1-\frac{a(b+c)}{a^2+bc})\geq 0< = > \frac{(a-b)(a-c)}{a^2+bc}+\frac{(b-c)(b-a)}{b^2+ac}+\frac{(c-a)(c-b)}{c^2+ab}\geq 0$

Nhưng bđt này luôn đúng vì đây là Schur mở rộng 




#482913 $\boxed{\text{Chuyên Đề}}$ Bất đẳng thức - Cực trị

Đã gửi bởi Hoang Tung 126 on 13-02-2014 - 18:25 trong Bất đẳng thức và cực trị

52) Cho $\left\{\begin{matrix}a;b;c>0 & & \\ a+b+c=4 & & \end{matrix}\right.$. Cmr: $4\leq \sum \sqrt{a+b}\leq 2\sqrt{6}$

 

53) Cho $a;b;c$ là độ dài 3 cạnh một tam giác, Cmr: $\sum \sqrt{a+b-c}\leq \sum \sqrt{a}$

 

54) Cho $a;b;c$ là độ dài 3 cạnh một tam giác có chu vi bằng $2$. Cmr: $\sum a^2<2(1-abc)$

 

55) Xác định dạng tam giác $ABC$ có chu vi bằng $1$ biết $A=\sum \frac{a}{1-2a}$ đạt $min$.

 

56) Cho $a;b;c>0$ thỏa $a^2+b^2+c^2=1$. Cmr: $\sum \frac{a^2}{1+b-a}\geq 1$

 

57) Cho $a;b;c$ là độ dài 3 cạnh một tam giác. Cmr: $\sum \sqrt[3]{a+b-c}\leq \sum \sqrt[3]{a}$

Bài 54:Theo bdt Tam giác có:$1=a+b+c> a+a=2a= > 1-2a> 0$.Tương tự $1-2b> 0,1-2c> 0$

Nhân theo vế $= > (1-2a)(1-2b)(1-2c)> 0< = > 4(\sum ab)> 2\sum a+8abc-1=1+8abc= > 2(\sum a)^2> 2\sum a^2+8abc+1= > \sum a^2+4abc< 1$(DPCM)




#482919 $\boxed{\text{Chuyên Đề}}$ Bất đẳng thức - Cực trị

Đã gửi bởi Hoang Tung 126 on 13-02-2014 - 18:32 trong Bất đẳng thức và cực trị

52) Cho $\left\{\begin{matrix}a;b;c>0 & & \\ a+b+c=4 & & \end{matrix}\right.$. Cmr: $4\leq \sum \sqrt{a+b}\leq 2\sqrt{6}$

 

53) Cho $a;b;c$ là độ dài 3 cạnh một tam giác, Cmr: $\sum \sqrt{a+b-c}\leq \sum \sqrt{a}$

 

54) Cho $a;b;c$ là độ dài 3 cạnh một tam giác có chu vi bằng $2$. Cmr: $\sum a^2<2(1-abc)$

 

55) Xác định dạng tam giác $ABC$ có chu vi bằng $1$ biết $A=\sum \frac{a}{1-2a}$ đạt $min$.

 

56) Cho $a;b;c>0$ thỏa $a^2+b^2+c^2=1$. Cmr: $\sum \frac{a^2}{1+b-a}\geq 1$

 

57) Cho $a;b;c$ là độ dài 3 cạnh một tam giác. Cmr: $\sum \sqrt[3]{a+b-c}\leq \sum \sqrt[3]{a}$

Bài 57:Đặt $\sqrt[3]{b+c-a}=x,\sqrt[3]{a+c-b}=y,\sqrt[3]{a+b-c}=z= > a=\frac{y^3+z^3}{2},b=\frac{x^3+z^3}{2},c=\frac{x^3+y^3}{2}$

BĐT $< = > \sum \sqrt[3]{\frac{y^3+x^3}{2}}\geq \sum x$

Theo AM-GM có:$y^3+x^3\geq \frac{(y+x)^3}{4}= > \sum \sqrt[3]{\frac{y^3+x^3}{2}}\geq \sum \frac{x+y}{2}=\sum x$




#482181 $\boxed{\text{Chuyên Đề}}$ Bất đẳng thức - Cực trị

Đã gửi bởi Hoang Tung 126 on 09-02-2014 - 13:09 trong Bất đẳng thức và cực trị

Bài 5:Theo AM-GM có:$\sum \sqrt{\frac{b+c}{a}.1}\leq \frac{1}{2}\sum (\frac{b+c}{a}+1)=\frac{1}{2}\sum \frac{a+b+c}{a}= > \sum \sqrt{\frac{a}{b+c}}\geq \frac{2\sum a}{\sum a}=2$

 Dấu = xảy ra tại a=0,b=c




#482179 $\boxed{\text{Chuyên Đề}}$ Bất đẳng thức - Cực trị

Đã gửi bởi Hoang Tung 126 on 09-02-2014 - 13:05 trong Bất đẳng thức và cực trị

Bài 4:Theo AM-GM có:$A=\frac{(x+y+z)(x+y)}{xyzt}=\frac{(x+y+z+t)^2(x+y+z)(x+y)}{4xyzt}\geq \frac{4t(x+y+z).(x+y+z)(x+y)}{xyzt}=\frac{4t(x+y+z)^2(x+y)}{xyzt}\geq \frac{4t.4z(x+y).(x+y)}{xyzt}=\frac{16tz(x+y)^2}{xyzt}\geq \frac{16tz.4xy}{xyzt}=64= > P\geq 64$




#482725 $\boxed{\text{Chuyên Đề}}$ Bất đẳng thức - Cực trị

Đã gửi bởi Hoang Tung 126 on 12-02-2014 - 16:51 trong Bất đẳng thức và cực trị

Bài 45:Ta có:$a+b=a^2-ab+b^2=\frac{3}{4}(a-b)^2+\frac{1}{4}(a+b)^2\geq \frac{1}{4}(a+b)^2= > 4(a+b)\geq (a+b)^2= > a+b\leq 4$(Do $a+b\geq 0$)

$= > Q=a^3+b^3=(a+b)(a^2-ab+b^2)=(a+b)(a+b)\leq 4.4=16$

Đẳng thức xảy ra tại a=b=2




#482238 $\boxed{\text{Chuyên Đề}}$ Bất đẳng thức - Cực trị

Đã gửi bởi Hoang Tung 126 on 09-02-2014 - 16:02 trong Bất đẳng thức và cực trị

Bài 11:Ta có:$A=2\sum ab+\sum \frac{1}{ab}=2\sum ab+\frac{\sum a}{abc}=2\sum ab+\frac{3}{abc}=\sum ab+\sum ab+\frac{3}{abc}\geq 3\sqrt[3]{\frac{(\sum ab)^2}{abc}}\geq 3\sqrt[3]{\frac{3abc(\sum a).3}{abc}}=3\sqrt[3]{9(\sum a)}=3\sqrt[3]{3.9}=9$

Dấu = xảy ra tại a=b=c=1




#482178 $\boxed{\text{Chuyên Đề}}$ Bất đẳng thức - Cực trị

Đã gửi bởi Hoang Tung 126 on 09-02-2014 - 13:03 trong Bất đẳng thức và cực trị

Giờ sẽ là BĐT và Cực Trị nhé.

1) (BĐT Schur)

Cmr: $\sum a^3+3abc\geq \sum ab(a+b)$

2) Tìm Min; Max của $A=xy$ biết $x;y$ nguyên dương và $x+y=2005$.

3) Tìm Min $A=|11^m-5^n|$ với $m;n$ nguyên dương.

4) Cho $x;y;z;t$ dương thỏa $x+y+z+t=2$

Tìm Min $A=\frac{(x+y+z)(x+y)}{xyzt}$

5) Cho $a;b;c>0$. Cmr: $\sum \sqrt{\frac{a}{b+c}}\geq 2$

6) Cho $a;b;c>0$. Cmr: $\sum \sqrt{\frac{a(b+c)}{a^2+bc}}$

Bài 1:BĐT $< = > x(x-y)(x-z)+y(y-z)(y-x)+z(z-x)(z-y)\geq 0$

Không mất tổng quát giả sử $x\leq y\leq z= > z(z-x)(z-y)\geq 0$

Do đó ta cần CM :$y(y-z)(y-x)+x(x-z)(x-y)\geq 0< = > (x-y)(x^2-xz-y^2+yz)\geq 0< = > (x-y)((x-y)(y+x)-z(x-y))\geq 0< = > (x-y)^2(x+y-z)\geq 0$(Luôn đúng do $x\geq y\geq z$)




#482550 $\boxed{\text{Chuyên Đề}}$ Bất đẳng thức - Cực trị

Đã gửi bởi Hoang Tung 126 on 11-02-2014 - 17:00 trong Bất đẳng thức và cực trị

Bài 24: Theo Bunhiacopkxi có:$\sum \frac{a^2}{1+b-a}=\sum \frac{a^4}{a^2+a^2b-a^3}\geq \frac{(\sum a^2)^2}{\sum a^2+\sum a^2b-\sum a^3}\geq \frac{(\sum a^2)^2}{\sum a^2}=\sum a^2=1$

(Do áp dụng bdt AM-GM có:$\sum a^3\geq \sum a^2b$)




#482724 $\boxed{\text{Chuyên Đề}}$ Bất đẳng thức - Cực trị

Đã gửi bởi Hoang Tung 126 on 12-02-2014 - 16:48 trong Bất đẳng thức và cực trị

Bài 44:Theo Bunhiacopxki co:$\sum \sqrt{2x^2+xy+y^2}=\sum \sqrt{x^2+\frac{3}{4}(x+y)^2+\frac{1}{4}(x-y)^2}\geq \sum \sqrt{x^2+\frac{3(x+y)^2}{4}}=\frac{1}{2}\sum \sqrt{4x^2+3(x+y)^2}=\frac{1}{2}\sum \sqrt{(2x)^2+(x+y)^2+(x+y)^2+(x+y)^2}\geq \frac{1}{2}\sum \frac{2x+x+y+x+y+x+y}{4}=\frac{1}{2}.\sum \frac{5x+3y}{4}=\sum x=1$




#482722 $\boxed{\text{Chuyên Đề}}$ Bất đẳng thức - Cực trị

Đã gửi bởi Hoang Tung 126 on 12-02-2014 - 16:40 trong Bất đẳng thức và cực trị

Bài 43:Ta có:$\sum \sqrt{x^2+xy+y^2}=\sum \sqrt{\frac{3(x+y)^2}{4}+\frac{(x-y)^2}{4}}\geq \sum \sqrt{\frac{3(x+y)^2}{4}}=\sum \frac{\sqrt{3}(x+y)}{2}=\sqrt{3}(\sum x)=\sqrt{3}.1=\sqrt{3}$

 Đẳng thức xảy ra tại $x=y=z=\frac{1}{3}$




#567714 Topic tổng hợp một số bất đẳng thức trong kì thi MO các nước

Đã gửi bởi Hoang Tung 126 on 23-06-2015 - 20:07 trong Bất đẳng thức - Cực trị

Bài 144(Poland MO): Cho a,b,c>0. CMR: nếu ab+bc+ca=1 thì

$\frac{(a^2+1)^2}{bc(b+c)}+\frac{(b^2+1)^2}{ca(c+a)}+\frac{(c^2+1)^2}{ab(a+b)}\geq 8(\sqrt{ab}+\sqrt{bc}+\sqrt{ca})$

Ta có : $a^2+1=a^2+ab+bc+ac=(a+b)(a+c)$

 

Theo Cosi và BDT  $(x+y)(y+z)(x+z)\geq \frac{8(\sum x)(\sum xy)}{9}$$(x+y)^3\geq 4xy(x+y)$ ta có :

 

  $\sum \frac{(a^2+1)^2}{bc(b+c)}=\sum \frac{\left [ (a+b)(a+c) \right ]^2}{bc(b+c)}\geq 3\sqrt[3]{\frac{(a+b)^4(b+c)^4(c+a)^4}{(abc)^2(a+b)(b+c)(c+a)}}=3\sqrt[3]{\frac{(a+b)^3(b+c)^3(c+a)^3}{(abc)^2}}\geq 3\sqrt[3]{\frac{4ab(a+b).4bc(b+c).4ac(a+c)}{(abc)^2}}=3\sqrt[3]{64(a+b)(b+c)(c+a)}\geq 12\sqrt[3]{\frac{8(\sum a)(\sum ab)}{9}}=12\sqrt[3]{\frac{8(\sum a).1}{9}}\geq 12\sqrt[3]{\frac{8\sqrt{3(\sum ab)}}{9}}=12\sqrt[3]{\frac{8\sqrt{3}}{9}}=24\sqrt[3]{\frac{1}{\sqrt{27}}}=\frac{24}{\sqrt{3}}=8\sqrt{3}= > \sum \frac{(a^2+1)^2}{bc(b+c)}\geq 8\sqrt{3}$   (1)

 

 Theo Bunhia có :$8(\sum \sqrt{ab})\leq 8\sqrt{3(\sum ab)}=8\sqrt{3.1}=8\sqrt{3}$  (2)

 

  Từ (1),(2) $= > \sum \frac{(a^2+1)^2}{bc(b+c)}\geq 8(\sum \sqrt{ab})$

 

  Dấu = xảy ra khi $a=b=c=\frac{1}{\sqrt{3}}$




#570265 Topic tổng hợp một số bất đẳng thức trong kì thi MO các nước

Đã gửi bởi Hoang Tung 126 on 06-07-2015 - 20:28 trong Bất đẳng thức - Cực trị

 Dạo này TOPIC ít người tham gia vậy ,vẫn những bài toán từ hôm trước




#564569 Topic tổng hợp một số bất đẳng thức trong kì thi MO các nước

Đã gửi bởi Hoang Tung 126 on 09-06-2015 - 11:22 trong Bất đẳng thức - Cực trị

Bài 93 ( IMO 2012 ) : Cho $a_{2},a_{3},...,a_{n}$ thỏa mãn : $a_{2}.a_{3}...a_{n}=1$ . Chứng minh rằng : 

$(1+a_{2})^{2}(1+a_{3})^{3}...(1+a_{n})^{n}>n^{n}$

Theo Cosi ta có : 

 

  $1+a_{2}\geq 2\sqrt{a_{2}}= > (1+a_{2})^2\geq 2^2.a_{2}$

 $a_{3}+1=a_{3}+\frac{1}{2}+\frac{1}{2}\geq 3\sqrt[3]{\frac{a_{3}}{2^2}}= > (1+a_{3})^3\geq \frac{3^3.a_{3}}{2^2}$

  .....................................................................................................

 $(a_{n}+1)=a_{n}+\frac{1}{n-1}+\frac{1}{n-1}+...+\frac{1}{n-1}\geq n\sqrt[n]{\frac{a_{n}}{(n-1)^{n-1}}}= > (1+a_{n})^n\geq n^{n}.\frac{a_{n}}{(n-1)^{(n-1)}}$

 

 Nhân theo vế các bất đẳng thức 

 

 $= > (1+a_{2})^2(1+a_{3})^3...(1+a_{n})^n\geq 2^{2}.3^{3}...n^{n}.(a_{1}a_{2}...a_{n}).(\frac{1}{2^{2}}.\frac{1}{3^{3}}...\frac{1}{(n-1)^{n-1}})=n^{n}= > (1+a_{2})^2(1+a_{3})^3...(1+a_{n})^{n}\geq n^{n}$

      (Do $a_{2}a_{3}...a_{n}=1$)

 

 Dấu = xảy ra khi $a_{2}=1,a_{3}=\frac{1}{2},...a_{n}=\frac{1}{n-1}= > a_{2}a_{3}...a_{n}=\frac{1}{2.3...(n-1)}$

 

Nhưng điều này vô lý do $a_{2}a_{3}...a_{n}=1$

 

  Do đó dấu = ko xảy ra ,tức là $(a_{2}+1)^2(a_{3}+1)^3...(a_{n}+1)^{n}> n^{n}$




#561496 Topic tổng hợp một số bất đẳng thức trong kì thi MO các nước

Đã gửi bởi Hoang Tung 126 on 25-05-2015 - 11:57 trong Bất đẳng thức - Cực trị

Bài 53 : ( Mediterranean MO 2002 ) Cho $a,b,c$ không âm thỏa mãn $a^{2}+b^{2}+c^{2}=1$ . CMR : 

$\frac{a}{b^{2}+1}+\frac{b}{c^{2}+1}+\frac{c}{a^{2}+1}\geq \frac{3}{4}(a\sqrt{a}+b\sqrt{b}+c\sqrt{c})^{2}$

 

Bài 53: Theo Bunhiacopxki ta có :

 

 $\frac{a}{b^2+1}+\frac{b}{c^2+1}+\frac{c}{a^2+1}=\frac{a^3}{a^2b^2+a^2}+\frac{b^3}{b^2c^2+b^2}+\frac{c^3}{c^2a^2+c^2}\geq \frac{(a\sqrt{a}+b\sqrt{b}+c\sqrt{c})^2}{(a^2+b^2+c^2)+(a^2b^2+b^2c^2+c^2a^2)}\geq \frac{(a\sqrt{a}+b\sqrt{b}+c\sqrt{c})^2}{a^2+b^2+c^2+\frac{(a^2+b^2+c^2)^2}{3}}=\frac{(a\sqrt{a}+b\sqrt{b}+c\sqrt{c})^2}{1+\frac{1}{3}}=\frac{3}{4}(a\sqrt{a}+b\sqrt{b}+c\sqrt{c})^2$

 

 Dấu = xảy ra khi $a=b=c=\frac{1}{\sqrt{3}}$




#560153 Topic tổng hợp một số bất đẳng thức trong kì thi MO các nước

Đã gửi bởi Hoang Tung 126 on 18-05-2015 - 14:21 trong Bất đẳng thức - Cực trị

 

Câu 30:(Bosnia 2008)

Cho $x,y,z$ là các số thực.CMR

$x^2+y^2+z^2-xy-yz-zx\geq max\left \{ \frac{3}{4}(x-y)^2,\frac{3}{4}(y-z)^2,\frac{3}{4}(z-x)^2 \right \}$

Ta sẽ chứng minh : 

 

  $x^2+y^2+z^2-xy-yz-xz\geq \frac{3}{4}(x-y)^2< = > 4(x^2+y^2+z^2-xy-yz-xz)\geq 3(x-y)^2< = > x^2+y^2+4z^2+2xy-4yz-4xz\geq 0< = > (x+y-2z)^2\geq 0$  (Luôn đúng)

 

Lập luận tương tự $x^2+y^2+z^2-xy-yz-xz\geq \frac{3}{4}(y-z)^2$

 

                                $x^2+y^2+z^2-xy-yz-xz\geq \frac{3}{4}(x-z)^2$

 

Từ đó $= > x^2+y^2+z^2-xy-xz-yz\geq max{\frac{3}{4}(x-y)^2,\frac{3}{4}(y-z)^2,\frac{3}{4}(x-z)^2}$




#561498 Topic tổng hợp một số bất đẳng thức trong kì thi MO các nước

Đã gửi bởi Hoang Tung 126 on 25-05-2015 - 12:13 trong Bất đẳng thức - Cực trị

 

Bài 54 : ( IMO shortlist 2006 ) Cho $a,b,c$ là ba cạnh của tam giác . CMR : 

$\sum \frac{\sqrt{b+c-a}}{\sqrt{b}+\sqrt{c}-\sqrt{a}}\leq 3$

  Đặt $\sqrt{a}=x,\sqrt{b}=y,\sqrt{c}=z$

 

Từ đó $\sum \frac{\sqrt{b+c-a}}{\sqrt{b}+\sqrt{c}-\sqrt{a}}=\sum \frac{\sqrt{y^2+z^2-x^2}}{y+z-x}\leq \sqrt{3(\sum \frac{y^2+z^2-x^2}{(y+z-x)^2})}\leq 3< = > \sum \frac{y^2+z^2-x^2}{(y+z-x)^2}\leq 3< = > \sum (\frac{y^2+z^2-x^2}{(y+z-x)^2}-1)\leq 0< = > \sum \frac{(y^2+z^2-x^2)-(y+z-x)^2}{(y+z-x)^2}\leq 0< = > \sum \frac{-2x^2-2yz+2xz+2xy}{(y+z-x)^2}\leq 0< = > \sum \frac{(x-y)(x-z)}{(y+z-x)^2}\geq 0$

$< = > \frac{(x-y)(x-z)}{(y+z-x)^2}+\frac{(y-z)(y-x)}{(x+z-y)^2}+\frac{(z-x)(z-y)}{(x+y-z)^2}\geq 0$  (1)

 

   Không mất tổng quát giả sử $x\geq y\geq z> 0= > \frac{(z-x)(z-y)}{(x+y-z)^2}\geq 0$

 

Ta cần CM :$\frac{(x-y)(x-z)}{(y+z-x)^2}+\frac{(y-z)(y-x)}{(x+z-y)^2}\geq 0< = > (x-y)(\frac{x-z}{(y+z-x)^2}-\frac{y-z}{(x+z-y)^2})\geq 0< = > \frac{x-z}{(y+z-x)^2}-\frac{y-z}{(x+z-y)^2}\geq 0$  

(Do $x-y\geq 0$)

 

 BDT trên đúng do $x\geq y= > \frac{x-z}{(y+z-x)^2}\geq \frac{y-z}{(y+z-x)^2}= > \frac{x-z}{(y+z-x)^2}-\frac{y-z}{(x+z-y)^2}\geq (y-z)(\frac{1}{(y+z-x)^2}-\frac{1}{(x+z-y)^2}=\frac{(y-z)}{(y+z-x)^2(x+z-y)^2}((x+z-y)^2-(y+z-x)^2)=\frac{4z(y-z)(x-y)}{(y+z-x)^2(x+z-y)^2}\geq 0$

 (Đúng do $x\geq y\geq z= > (y-z)(x-y)\geq 0$)

 

  Do đó ta có ĐPCM .Dấu = xảy ra khi $x=y=z< = > a=b=c$




#564464 Topic tổng hợp một số bất đẳng thức trong kì thi MO các nước

Đã gửi bởi Hoang Tung 126 on 08-06-2015 - 19:43 trong Bất đẳng thức - Cực trị

Bài 89: (Turkish MO 2007)

Cho $a,b,c$ thực dương thỏa mãn: $a+b+c=3$.Chứng minh bất đẳng thức:

 

$\frac{a^2+3b^2}{ab^2(4-ab)}+\frac{b^2+3c^2}{bc^2(4-bc)}+\frac{c^2+3a^2}{ca^2(4-ca)}\geq 4$

Theo Cosi ta có :

 

 $\sum \frac{a^2+3b^2}{ab^2(4-ab)}=\sum \frac{(a^2+b^2)+2b^2}{ab^2(4-ab)}\geq \sum \frac{2ab+2b^2}{ab^2(4-ab)}=\sum \frac{2b(a+b)}{ab^2(4-ab)}$

$=2\sum \frac{a+b}{ab(4-ab)}\geq 2\sum \frac{2\sqrt{ab}}{ab(4-ab)}=4\sum \frac{1}{\sqrt{ab}(4-ab)}$   (1)

 

Mà $8+ab=(4-ab)+(4-ab)+3ab\geq 3\sqrt[3]{3ab(4-ab)^2}= > (8+ab)^3\geq 81ab(4-ab)^2$

$= > ab(4-ab)^2\leq \frac{(8+ab)^3}{81}= > \sqrt{ab}(4-ab)\leq \frac{\sqrt{(8+ab)^3}}9{}= > \frac{1}{\sqrt{ab}(4-ab)}\geq \frac{9}{\sqrt{(8+ab)^3}}$

$= > \sum \frac{1}{\sqrt{ab}(4-ab)}\geq 9\sum \frac{1}{\sqrt{(8+ab)^3}}\geq 9.3\sqrt[3]{\frac{1}{\sqrt{(8+ab)^3(8+bc)^3(8+ac)^3}}}=\frac{27}{\sqrt{(8+ab)(8+bc)(8+ac)}}\geq \frac{27}{\sqrt{\frac{(8+ab+8+bc+8+ac)^3}{27}}}\geq \frac{27}{\sqrt{\frac{(24+\frac{(a+b+c)^2}{3})}{27}}}=\frac{27}{\sqrt{\frac{(25+3)^3}{27}}}=\frac{27}{\sqrt{27^2}}=1$

   $= > \sum \frac{1}{\sqrt{ab}(4-ab)}\geq 1$    (2)

 

   Từ (1) ,(2) $= > \sum \frac{a^2+3b^2}{ab^2(4-ab)}\geq 4$ và ta có ĐPCM

 

Dấu = xảy ra khi $a=b=c=1$